Sunday 17 July 2022

NUMBER SERIES

 

MATHS & REASONING BY - DR V K OMAR

Directions(1-2): In each of the following questions a number series is given. Below the series, a number along with (1), (2), (3) (4) and (5) is given. You have to complete the series following the same sequence as that of the given series. Then answer the question that follows.

Q1. 5 , 7,  10,  36, 136,  690
       2 ,(1), (2) , (3),  (4),  (5)
What will come in place of (3)?
(a) 26
(b) 64
(c) 18
(d) 46
Ans.(c)
Sol.  The series is based on the following pattern:
5 × 1 + 2 = 7
7 × 2 – 4 = 10
10 × 3 + 6 = 36
36 × 4 – 8 = 136
136 × 5 + 10 = 690
Similarly, the new series is as follows:
2 × 1 + 2 = 4 …. (1)
4 × 2 – 4 = 4 …. (2)
4 × 3 + 6 = 18 …. (3)
Therefore, the number 18 will come in place of (3).

Q2. 8,    4,    6,   15,   52.5,   236.25
       4,  (1),  (2), (3),    (4),      (5)
What will come in place of (4)?
(a) 36.25 
(b) 26.25 
(c) 16.25
(d) 32.75 
Ans.(b)
Sol.  The series is based on following pattern:
8 × 0.5 = 4,
4 × 1.5 = 6,
6 × 2.5 = 15,
15 × 3.5 = 52.6,
52.5 × 4.5 = 236.25
Similarly,
4 × 0.5 = 2 …. (1)
2 × 1.5 = 3 …. (2)
3 × 2.5 = 7.5 …. (3)
7.5 × 3.5 = 26.25 . …(4)
Therefore, the number 26.25 will come in place of (4).

Directions(3-5): In each of the following questions a number is missing. Find this missing number from the following alternatives.

Q3. 512, 49, 216, 25, ?
(a) 8
(b) 27 
(c) 64
(d) 36
Ans.(c)          
Sol.

512,     49,      216,     25,         64
(83),   (72),    (63),     (52),      (43)

Q4. 15, 24, 42, 69, (?), 150 
(a) 114
(b) 105
(c) 100
(d) 140
S4. Ans.(b)
Sol.  The pattern of the number series is as given below:
15 + 9 = 24
24 + 18 (= 9 + 9) = 42
42 + 27 (= 18 + 9) = 69
69 + 36 (= 27 + 9) = 105
105 + 45 (= 36 + 9) = 150

Q5. 3, 7, 16, 32, ?, 93 
(a) 53
(b) 57
(c) 54
(d) 60
Ans.(b)
Sol. 
Number Series: Reasoning Questions for SSC CGL Exam 2017_50.1
= 32 + 52
= 32 + 25 = 57 

Directions(6-10): In these questions, which of the following number complete the given sequence?

Q6.  8, 24, 12, 36, 18, 54, (….)
(a) 27          
(b) 108
(c) 68          
(d) 72
Ans. (a)
Sol. Numbers are alternatively multiplied by 3 and divided by 2.
So, the next number = 54 ÷ 2 = 27.

Q7.  2, 6, 12, 20, 30, 42, 56, (….)
(a) 61          
(b) 64
(c) 72          
(d) 70
Ans.(c)
Sol. The pattern is 1 x 2, 2 x 3, 3 x 4, 4 x 5, 5 x 6, 6 x 7, 7 x 8.
So, the next number is 8 x 9 = 72.

Q8.  4, -8, 16, -32, 64, (….)
(a) 128        
(b) -128
(c) 192        
(d) -192
Ans.(b)
Sol. Each number is the proceeding number multiplied by -2.
So, the required number is -128.

Q9. 122 , 170 , 290 , ?
(a) 369 
(b) 255 
(c) 432
(d) 362
Ans.(d)
Sol.  (11)2 = 121+1= 122,
(13)2= 169+1= 170
(17)2= 289 + 1= 290,
Similarly
(19)2 = 361 + 1 = 362

Q10. 2 ,3 , 8 , 63, ?
(a) 3900
(b) 3636
(c) 3968
(d) 3988
Ans.(c)
Sol. 22= 4-1= 3
32= 9-1 = 8
82= 64-1= 63
632=3969-1 = 3968 

Direction (11-13): In each of these questions, a number series is given. In each series, only one number is wrong. Find out the wrong number from the given alternatives?

Q11. 32, 16, 24, 65, 210, 945, 5197.5
(a) 16
(b) 210
(c) 65
(d)5197.5
Ans.(c)
Sol.
Number Series: Reasoning Questions for SSC CGL Exam 2017_70.1 
So, wrong number is 65, it must be replaced by 60.

Q12. 6, 91, 584, 2935, 11756, 35277, 70558
(a) 35277
(b) 584
(c) 2935
(d) 91
Ans.(b)
Sol.  The pattern of number series is as follows:
6 × 7 + 72 = 42 + 49 = 91
91 × 6 + 62 = 546 + 36 = 582  [584]
582 × 5 + 52 = 2910 + 25 = 2935
2935 × 4 + 42 = 11740 + 16 = 11756
11756 × 3 + 32 = 35268 + 9 = 35277
So, wrong number is 584, it must be replaced by 582.

Q13. 3601, 3602, 1803, 604, 154, 36, 12
(a) 3601 
(b) 154
(c) 604
(d) 1803
Ans.(b)
Sol.
Number Series: Reasoning Questions for SSC CGL Exam 2017_80.1
So, wrong number is 154, it must be replaced by 155.


Directions(14-15): What should come in place of question mark (?) in the following number series?

Q14. 656, 352, 200, 124, 86, (?)
(a) 97
(b) 49 
(c) 67
(d) 57
Ans.(c)
Sol.  The pattern of the number series is as given below:
Number Series: Reasoning Questions for SSC CGL Exam 2017_100.1
Missing term is 67.

Q15. 4.5, 18,  2.25,  ? , 1.6875, 33.75
(a) 35
(b) 25.5 
(c) 27
(d) 43 
Ans.(c)
Sol.  The series is
× 4, ÷ 8, × 12, ÷ 16, × 20

MATHS & REASONING BY - DR V K OMAR

Directions (1-7): In each of the following questions, a number series is given with one term missing. Choose the correct alternative that will continue the same pattern and replace the question mark in the given series. 

Q1. 120, 99, 80, 63, 48, ? 
(a) 35
(b) 36
(c) 37
(d) 38
Ans.(a)
Sol. The pattern is -21, – 19, – 17, – 15, …
So, missing term = 48 – 13 = 35 

Q2. 6, 17, 39, 72, ? 
(a) 89
(b) 94
(c) 116
(d) 123
Ans.(c)
Sol. The pattern is + 11, + 22, + 33, … 
So, missing term = 72 + 44 = 116 

Q3. 1, 9, 25, 49, 81, ? 
(a) 169
(b) 112
(c) 121
(d) 144 
Ans.(c)
Sol. The series consists of squares of consecutive odd numbers i.e. 12, 32, 52, 72, 92, ….
So, missing term = 112 = 121
Q4. 4, 6, 9, 13(1/2), ?   
(a) 17(1/2)
(b) 19
(c) 20(1/4)
(d) 22(3/4)
Ans.(c)
Sol. The pattern is ×3/2. So, missing number =13(1/2)×3/2=27/2×3/2=81/4=20(1/4)
Q5. 0, 2, 3, 5, 8, 10, 15, 17, 24, 26, ?
(a) 28
(b) 31
(c) 32
(d) 35
Ans.(d)
Sol. The given sequence is a combination of two series: 
I. 0, 3, 8, 15, 24, ?  and  II. 2, 5, 10, 17, 26 
The pattern in each one of I and II is +3, +5, +7, +9, ….
So, missing term = 24 + 11 = 35 

Q6. 2, 1, 2, 4, 4, 5, 6, 7, 8, 8, 10, 11, ? 
(a) 9
(b) 10
(c) 11
(d) 12
Ans.(b)
Sol. The given sequence is a combination of three series: 
I. 1st, 4th, 7th, 10th, 13th, terms i.e. 2, 4, 6, 8, ?
II. 2nd, 5th, 8th, 11th terms i.e. 1, 4, 7, 10 
III. 3rd, 6th, 9th, 12th terms i.e. 2, 5, 8, 11 
Clearly I consists of consecutive even numbers. So, the missing term is 10.  

Q7. 8, 9, 8, 7, 10, 9, 6, 11, 10, ?, 12 
(a) 5
(b) 7
(c) 8
(d) 11
Ans.(a)
Sol. The given sequence is a combination of three series:  
I. 1st, 4th, 7th, 10th terms i.e. 8, 7, 6, ? 
II. 2nd, 5th, 8th, 11th terms i.e. 9, 10, 11, 12 
III. 3rd, 6th, 9th terms i.e. 8, 9, 10  
The pattern in I is -1. So, missing term = 6 – 1 = 5. 

Directions (8): In this question, one term in the number series is wrong. Find out of the wrong term. 
Q8. 46080, 3840, 348, 48, 8, 2, 1 
(a) 48
(b) 348
(c) 24
(d) 2
Ans.(b)
Sol. The correct patter is ÷ 12, ÷ 10, ÷ 8, ÷ 6, ….
So, 348 is wrong and must be replaced by (3840 ÷ 10) i.e. 384. 

Directions (9-15): In each of the following questions, various terms of an alphabet series are given with one or more terms missing as shown by (?). Choose the missing terms out of the given alternatives. 
Q9. B, D, F, I, L, P, ? 
(a) R
(b) S
(c) T
(d) U
Ans.(c)
Sol. 
Series Questions for SSC CGL Exam_50.1
Q10. Y, B, T, G, O, ? 
(a) N
(b) M
(c) L
(d) K
Ans.(c)
Sol. The given sequence is a combination of two series: 
I. Y, T, O and  II. B, G, ? 
I. Consist of 2nd, 7th and 12th letters from the end of the English alphabet, 
while II consists of 2nd, 7th and 12th letters from the beginning of the English alphabet. 
So, the missing letter in II is the 12th letter from the beginning of the English alphabet, which is L.
Q11. AZ, GT, MN, ?, YB
(a) JH
(b) SH
(c) SK
(d) TS
Ans.(b)
Sol. 
Series Questions for SSC CGL Exam_60.1
  
Q12. 3F, 6G, 11I, 18L, ?  
(a) 27O
(b) 25N
(c) 25P
(d) 27P
Ans.(d)
Sol. 
Series Questions for SSC CGL Exam_70.1
Q13. N5V, K7T, ?, E14P, B19N 
(a) H9R
(b) H10Q
(c) H10R
(d) I10R 
Ans.(c)
Sol. 
Series Questions for SSC CGL Exam_80.1

Q14. GH, JL, NQ, SW, YD, ? 
(a) EJ
(b) FJ
(c) EL
(d) FL
Ans.(d)
Sol. 
Series Questions for SSC CGL Exam_90.1 

Q15. UPI, ?, ODP, MBQ, IAW
(a) SGL
(b) SHJ
(c) SIK
(d) SJM
Ans.(b)
Sol. 
Series Questions for SSC CGL Exam_100.1


MATHS & REASONING BY - DR V K OMAR

1. What should come at the place of question mark (?) in the following number series? 
509, 68, 429, 140, 365, ?


A. 194
B. 192
C. 196
D. 195
E. 197




2. What should come at the place of question mark (?) in the following number series? 
1, 12, 14, ?, 66, 142


A. 40
B. 50
C. 56
D. 42
E. 38



3. What should come at the place of question mark (?) in the following number series? 
2√3, 2√5, √30, √42, 2√14, ?


A. 6√2
B. √60
C. 5√3
D. 3√5
E. √50

4. What should come at the place of question mark (?) in the following number series? 
1, 2, 6, 3, 289, ?

A. 3172
B. 3414
C. 3192
D. 3429
E. 3439

5. What should come at the place of question mark (?) in the following number series?
8, 17, 34, 58, 88, ?

A. 130
B. 123
C. 102
D. 132
E. 120

Answers With Solution:

1. Ans. C.

Solution:



2. Ans. E.

Solution:



3. Ans. A.

Solution: 


4. Ans. B.

Solution:




5. Ans.B. 

Solution: 



MATHS & REASONING BY - DR V K OMAR

1. What should come at the place of question mark (?) in the following number series? 
2556, 636, 156, ?, 6
A. 36 
B. 72 
C. 100 
D. 112 
E. 20 

2. What should come at the place of question mark (?) in the following number series? 
21, 85, 253, 509, ? 
A. 506 
B. 505 
C. 510 
D. 521 
E. 511 

3. What should come at the place of question mark (?) in the following number series? 
19, 33, 67, 97, 147, ? 
A. 193 
B. 200 
C. 210 
D. 205 
E. 199 

4. What should come at the place of question mark (?) in the following number series? 
49, 60, 98, 102, 133, ? 
A. 139 
B. 145 
C. 140 
D. 130 
E. 120 

5. What should come at the place of question mark (?) in the following number series? 
318, 206, 150, ?, 108, 101 
A. 130 
B. 115 
C. 122 
D. 120 
E. 132 

Solution: 

1. Ans. A. 

Solution: 


2. Ans. B. 

Solution: 



3. Ans. A. 

Solution:
 

4. Ans. D. 

Solution: 



5. Ans. C. 

Solution: 

MATHS & REASONING BY - DR V K OMAR

Number Series Questions Asked- IPPB PO 2017



Ques 1) 5, 4, 7, 20, 79, ?
Ques 2)18, 9, 9, 13.5, 27, ?
Ques 3) 3, 83, 152, 209, 253, ?
Ques 4) 318, 160, 82, 44, ?
Ques 5) 500, 500, 493, 467, 404, ?
Ques 6) 22, 11, 11, 16.5, 33, ?
Ques 7) 6, 5, 9, 26, 103, ?
Ques 8) 401, 401, 394, 368, 305, ?
Ques 9) 4, 75, 136, 186, 224, ?
Ques 10)
158, 80, 42, 24, ?
Ques 11) 3, 72, 129, 173, 203, ?
Ques 12) 309, 309, 302, 276, 213, ?
Ques 13) 24, 12, 12, 18, 36, ?
Ques 14) 126, 64, 34, 20, ? ,12
Ques 15) 4, 3, 5, 14, 55, ?
1) 
⇒ 5 ✕ 1-1=4
4 ✕ 2-1=7
7 ✕ 3-1=20
20 ✕ 4-1=79
79 ✕ 5-1=394
Ans 394 

2)

⇒ 18 ✕ 0.5 = 9
⇒ 9 ✕ 1 = 9
⇒ 9 ✕ 1.5 = 13.5
⇒ 15.5 ✕ 2= 27
⇒ 27 ✕ 2.5 = 67.5 answers

3 ) 

There is pattern between the differences of the numbers given in the series.
83 - 3 = 80
152 - 83 = 69
209 - 152 = 57
253 -209 = 44

Now
⇒ 80 - 69 = -11
⇒ 69 - 57 = -12
⇒ 57 - 44 = -13
⇒ 44 - 30 = -14

30 is the answers

4 ) 

This questions was very easy, there was a clear pattern in series, no need to find the differences.
⇒ x0.5+1
⇒ x0.5+2
⇒ x0.5+3
⇒ x0.5+4

5 )

Again we need to find the differences between the numbers and then there was a pattern in the differences.
⇒ 0....-7....-26....-63....-124 
⇒ 7....19....37....61 
⇒12....18....24

6) 

⇒ x0.5....x1....x1.5....x2....x2.5
⇒ 33 ✕ 2.5 = 82.5

7)

⇒ x1-1….x2-1…x3-1….x4-1

8)

There is pattern between the differences of the numbers given in the series.
⇒ 0....-7....-26....-63....-124 
⇒ 7....19....37....61 
⇒ 12....18....24 

9) 

+71....+61....+50....+38....+25

-10....-11....-12....-13

10 ) 

x0.5+1….x0.5+2….x0.5+3….x0.5+4
 24 ✕ 1/2 + 3=
15 Answer

MATHS & REASONING BY - DR V K OMAR

Number Series Asked in IBPS RRB OS - V (Prelim) Exam - 06.11.2016 (1st Shift)



Directions (1-5): What should come in place of the question mark (?) in the following number series?

#1.

4, 16, 26, 34, 40, ? 
a) 45
b) 46
c) 43
d) 55
e) 44

#2.

825, 582, 501, 474, 465,? 
a) 484
b) 642
c) 246
d) 462
e) 532

#3.

8, 12, 24, 60, 180, ? 
a) 360
b) 300
c) 630
d) 600
e) 720

#4.

3, 4, 10, 33, 136, ? 
a) 586
b) 685
c) 569
d) 690
e) 265

#5.

16, 17, 13, 22, 6, ? 
a) 31
b) 2
c) 1
d) 12
e) 13


Solutions :



1. e) 44

Solution: 
4………16……….26………..34…...….40……… 
…….+12…..+10…..…..+8…..…...+6….…...+4 
Answer: 40 + 4 = 44 

2. d) 462

Solution: 
825…….…..582….…..501….…..474……..465……… 
……..-243…...…-81………-27…….…-9…….…-3… 
Answer: 465 – 3 = 462 

3. c) 630

Solution: 
8 x 1.5 = 12 
12 x 2 = 24 
24 x 2.5 = 60 
60 x 3 = 180 
180 x 3.5 = 630 

4. b) 685
Solution: 
3 x 1 + 1 = 4 
4 x 2 + 2 = 10 
10 x 3 + 3 = 33 
33 x 4 + 4 = 136 
136 x 5 + 5 = 685

5. a) 31

Solution: 
16………..17……..13……....22……..6…… 
…...+1…........-4……....+9…….-16……(+25) 
….(1 x 1)….(2 x 2)….(3x 3)….(4x4)....(5 x 5) 
Answer: 6+25= 31 

MATHS & REASONING BY - DR V K OMAR

Number Series Asked in IBPS RRB OS - V (Prelim) Exam - 05.11.2016 (1st Shift)






Directions (1-5): What should come in place of the question mark (?) in the following number series?

#1. 

6          4           5           11           39           ?
a) 188
b) 198
c) 189
d) 199
e) 200

#2. 

14          7         7          14          56           ?
a) 158
b) 184
c) 484
d) 884
e) 448

#3. 

2          3          7          16           32           ?
a) 57
b) 59
c) 56
d) 60
e) 75

#4. 

8          14           24          38           56           ?
a) 55
b) 65
c) 96
d) 78
e) 87

#5. 

5          7          13           25           45           ?
a) 57
b) 75
c) 55
d) 77
e) 54

Answers:


1. c) 189

Solution:
6 x 1 – 2 = 4
4 x 2 – 3 = 5
5 x 3 – 4 = 11
11 x 4 – 5 = 39
39 x 5 – 6 = 189

2. e) 448

Solution:
14 x 0.5 = 7
7 x 1 = 7
7 x 2 = 14
14 x 4 = 56
56 x 8 = 448

3. a) 57

Solution:
2 + 12 = 3
3 + 22 = 7
7 + 32 = 16
16 + 42 = 32
32 + 52 = 57

4. d) 78

Solution:
8……....14….....24….......38….....56
…..+6…...+10…....+14…......+18
…….+4……...+4…......…+4…….+4

Answer: 56 + 18 + 4 = 78

5. b) 75

Solution:
5….....…7……...13…......25….......45
…....+2….....+6…......+12..….+20
…......…+4…...…+6….......+8……+10

Answer: 45 + 20 + 10 = 75

MATHS & REASONING BY - DR V K OMAR


Number Series Asked in IBPS RRB OS - V (Prelim) Exam - 04.11.2016 (1st Shift)

Directions (1-5): What should come in place of the question mark (?) in the following number series?

#1. 

3          10           22           39           61           ?
a) 88
b) 85
c) 66
d) 95
e) 83


#2. 


9           14           21           32           45           ?
a) 52
b) 56
c) 72
d) 62
e) 66

#3. 


3           1.5           1.5           2.25           4.5           ?
a) 9.5
b) 11.25
c) 12.25
d) 10.45
e) 5.5

#4. 


2           4           11           37           153           ?
a) 771
b) 661
c) 551
d) 441
e) 221

#5. 


4           4           6           12           24          ?
a) 58
b) 42
c) 40
d) 42
e)  44

Answers:


1. a) 88

Trick: difference upto two lines

Solution:
3…...….10…...…22……....39…......61 
…..+7…….+12…….+17…….+22 
………+5…….+5……….+5 

Ans: 61 + 22+ 5 = 88
  
2. d) 62

Trick: difference of prime numbers 

Solution:
9….14…..21…...32…......45 
…+5…+7….+11......+13 

Ans: 45 + 17 = 62
  
3. b) 11.25 

Solution:
3 * 0.5 = 1.5 
1.5 * 1 = 1.5 
1.5 * 1.5 = 2.25 
2.25 * 2 = 4.5 
4.5 * 2.5 = 11.25 

4. a) 771

Solution:
2 * 1 + 2 =4 
4 * 2 + 3 = 11 
11 * 3 + 4= 37 
37 * 4 + 5 = 153 
153 * 5 + 6= 771
  
5. e) 44

Trick: difference upto two lines 

Solution:
4…........4......…6…......12.......…24
....+0….....+2........+6..........+12 
....…+2..........+4.........+6 

Ans: 24 + 12 + 8 = 44 

MATHS & REASONING BY - DR V K OMAR

Number Series Asked in IBPS PO - VI (Prelim) Exam - 27.10.2016 (1st Shift)Directions (1-5): What should come in place of the question mark (?) in the following number series?

#1. 

7, 6, 8, 15, 44, ?
a) 88
b) 120
c) 159
d) 150
e) 195

#2. 

16, 25, 42, 66, 96, ?
a) 120
b) 160
c) 118
d) 131
e) 130

#3. 

4, 6, 9, 34, 68, ?
a) 56
b) 64
c) 161
d) 68
e) 116


#4. 

7.6, 9.9, 14.5, 23.7, ?

a) 42.2
b) 42.1
c) 42.3
d) 42.4
e) 42.5

#5. 

9, 5, 4, 7.5, 13, ?
a) 35
b) 26
c) 34
d) 48
e) 62


Answers:


1. e) 195

Solution:
7 x 1 – 12 = 6
6 x 2 – 22 = 8
8 x 3 – 32 = 15
15 x 4 – 42 = 44
44 x 5 – 52 = 195

2. d) 131

Solution:
+9….…..+17…..…+24……...+30….…..+35
        +8…….…+7….…...+6…...…...+5


96 + 35 = 131


3. c) 161

Solution:
4 x 2 – 2 = 6
6 x 3 – 9 = 9
9 x 4 - 2 = 34
34 x 5 – 9 = 161

4. b) 42.1

Solution:
+2.3 ….. +4.6 ….. +9.2 .…. +18.4
7.6 + 2.3 = 9.9
9.9 + 4.6 = 14.5
14.5 + 9.2 = 23.7
23.7 + 18.4 = 42.1

5. a) 35

Solution:
9 x 0.5 + 0.5 = 5
5 x 1 – 1 = 4
4 x 1.5 + 1.5 = 7.5
7.5 x 2 – 2 = 13
13 x 2.5 + 2.5 = 35

MATHS & REASONING BY - DR V K OMAR

Number Series Asked in IBPS PO - VI (Prelim) Exam - 22.10.2016 (3rd Shift)

Directions (1-5): What should come in place of the question mark (?) in the following number series?

#1.

12, 6.5, 7.5, 17, 72, ?
a) 356
b) 82
c) 185
d) 292
e) 584

#2.

463, 220, 139, 112, ?, 100
a) 101
b) 100
c) 104
d) 103
e) 102


#3.

17, 30, 21, 34, 25, ?
a) 38
b) 30
c) 27
d) 36
e) 40

#4.

31, 33, 40, 68, 131, ?
a) 232
b) 257
c) 132
d) 250
e) 275

#5.

11, 16, 34, 66, 113, ?
a) 130
b) 160
c) 176
d) 175
e) 177

Answers:

1. e) 584
Solution:
12 x 0.5 + 0.5 = 6.5 
6.5 x 1 + 1 = 7.5 
7.5 x 2 + 2 = 17 
17 x 4 + 4 = 72 
72 x 8 + 8 = 584

2. d) 103 
Solution:
463 – 220 = 243 
220 – 139 = 81 
139 – 112 = 27 
112 – ? = 9 
? – 100 = 3 
So, ? = 112 – 9 = 103 

3. a) 38
Solution: 
17 +13 = 30 
30 – 9 = 21 
21 +13 = 34 
34 – 9 = 25 
25 + 13 = 38 

4. b) 257
Solution: 
31 + (13 + 1) = 31 + 2 = 33 
33 + (23 - 1) = 33 + 7 = 40 
40 + (33 + 1) = 40 + 28 = 68 
68 + (43 - 1) = 68 + 63 = 131 
131 + (53 + 1) = 131 + 126 = 257 

5. c) 176
Solution: 

MATHS & REASONING BY - DR V K OMAR


 I.Find the wrong number in the following series.


1.  40   326   2946   29418   323607

1.326
2.40
3.2946
4.323607
5.29418


2. 560   1089   1725   2443   3284   4245

1.2443
2.1725
3.4245
4.3284
5.560

3. 3252   3080   2958   2876   2826 

1.3080
2.2876
3.2826
4.3252
5.2958

4. 2442   1222   614   312   163   90   55.75

1.1222
2.312
3.90
4.614
5.163

5. 1250   1322   1452   1674   2024   2544

1.1322
2.1674
3.2544
4.2024
5.1250


II. Find the missing number in the given series.


6. 1953.125    781.25  312.5   ?   20 
1.48
2.49
3.50
4.51
5.52

7. 81   87  162   504   1992   9990   ?

1.59903
2.59906
3.56895
4.59904
5.59861

8.  49   72   118   ?    394   762   1498 

1.234
2.239
3.210
4.219
5.243

9.  142   143  156   193   272   417   ?

1.653
2.658
3.659
4.657
5.656

10.  118   122   158   206   356   536   ?

1.956
2.953
3.928
4.965
5.963

Solutions:

1.2946

40*8+6=326
326*9+7=294
2946*10+8=29418
29418*11+9=323607

2.1725

the difference between numbers is +23^2 ,+25^2 ,  +27^2 , +29^2 ....... 

3.3252

the difference between numbers is -(13^2+1), -(11^2+1), -(9^2+1) , -(7^2+1), -(5^2+1)

4. 90 

2442/2  + 1 = 1222
1222/2  + 3= 614
614/2   +  5  = 312
312/2   + 7 =163
163/2  + 9 = 89.5
89.5/2  + 11 = 55.75

5. 1250
  , 
the difference between numbers is +(3^3+3) , +(4^3+4) , +(5^3+5),  +(6^3+6) ,  +(7^3+7), 
+(8^3+8)


6.50

1953.125/2.5=781.25
781.25/2.5=312.5
312.5/2.5=125
125/2.5=50
50/2.5=20

7.59904

the difference between numbers is  *1+6 , *2-12, *3+18 , *4-24 ,*5+30 , *6-36

8.210 

the difference between numbers is  +23 , +46 , +92 , +184 , +368 ,+736

9.658

the difference between numbers is  +1^3+0 , +2^3+5 ,+3^3+10 , +4^3+15 , +5^3+20 ,+6^3+25

10. 928

the difference between numbers is 
+2^3-2^2
 +3^3-3^2 
 +4^3-4^2 
 +5^3-5^2 
 +6^3-6^2 
 +7^3-7^2  

MATHS & REASONING BY - DR V K OMAR

Number Series Asked in SBI PO Prelims Exam (Slot-I) - 02.07.2016

Directions (1-5): What should come in place of the question mark (?) in the following number series? 

1. 7        5        7        17        63       ?
a) 308
b) 302
c) 309
d) 409
e) 390

2. 50      ?     61      89      154       280
a) 52
b) 51
c) 60
d) 62
e) 60 

3. 17     19       25      37      ?       87
a) 47
b) 37
c) 57
d) 67
e) 75 

4. 11      14      19       28      43       ?
a) 55
b) 44
c) 77
d) 88
e) 66 

5. 26     144       590      1164      ?
a) 1296
b) 1182
c) 2059
d) 1182
e) 1181

Answers:


1. c) 309
Solution: 
7 x 1 – 2 = 5 
5 x 2 – 3 = 7 
7 x 3 – 4 = 17 
17 x 4 – 5 = 63 
63 x 5 – 6 = 309 

2. a) 52
Solution: 
50..50+(1^3+1) = 52 
52+(2^3+1)=61. 
61+(3^3+1)=89 
89+(4^3+1)=154 
154 + (5^3+1) = 280 

3. c) 57
Solution: 
17 + 1 x 2 = 19 
19 + 2 x 3 = 25 
25 + 3 x 4 = 37 
37 + 4 x 5 = 57 
57 + 5 x 6 = 87 

4. e) 66
Solution: 
3….5...9...15...23 
…2....4....6....8…
Answer 43+23= 66 

5. b) 1182
Solution: 
26 x 6 – 12 = 144 
144 x 4 + 14 = 590 
590 x 2 – 16 = 1164 
1164 x 1 + 18 = 1182

MATHS & REASONING BY - DR V K OMAR

Number Series Asked in SBI PO Prelims Exam (Slot-I) - 09.07.2016

Directions (1-5): What should come in place of the question mark (?) in the following number series? 
1. 150            102            70            46            26            ?
a) 16
b) 8
c) 10
d) 2
e) 4
2. 10            14            28            52            134            ?
a) 302
b) 268
c) 300
d) 304
e) 208 

3. 4500            900            90            6            ?            0.012
a) 0.3
b) 3
c) 3.33
d) 0.33
e) 3.3 

4. 24            11            10            14            27            ?
a) 66
b) 70.5
c) 68
d) 66.5
e) 68.5 

5. 8            7            12            33            128            ?
a) 528
b) 365
c) 653
d) 825
e) 635

Answers: 


1. b) 8

Solution:


2. d) 304

Solution:

10*3 - 2= 28
14*4 - 4= 52
28*5 - 6= 134
52*6- 8= 304 

3. a) 0.3

Solution:

4500/5 = 900
900/10 = 90
90/15 = 6
6/20 = 0.3
0.3/25 = 0.012 

4. d) 66.5

Solution:

24*(1/2) – 1 = 11
11*(2/2) – 1 = 10
10*(3/2) – 1 = 14
14*(4/2) – 1 = 27
27*(5/2) – 1 = 66.5 

5. e) 635

Solution:

8*1 – 1 = 7
7*2 – 2 = 12
12*3 – 3 = 33
33*4 – 4 = 128
128*5 – 5 = 635 

MATHS & REASONING BY - DR V K OMAR

Number Series Asked in RBI Grade B Manager (Phase-I) Exam - 04.09.2016 (1st Shift)

Directions (Q.1 -5): In the following number series only one number is wrong. Find out the wrong number. 

1. 18.3        20.6        16       22.9        13.7        2.2       11.4
a) 25.2
b) 18.3
c) 13.7
d) 22.9
e) 20.6

2. 9        5        6        10.5        23        61        183
a) 183
b) 10.5
c) 61
d) 5
e) 9 

3. 188        154        140       132       128       126       125
a) 125
b) 154
c) 132
d) 126
e) 188 

4. 2        4        11        37        151        771        4633
a) 11
b) 4633
c) 771
d) 151
e) 2 

5. 391        394        399       411        431       461        503
a) 503
b) 394
c) 399
d) 431
e) 391 


Solutions:

1. a) 25.2

18.3 + 2.3 = 20.6
20.6 - 4.6 = 16
16 + 6.9 = 22.9
22.9 - 9.2 = 13.7
13.7 +11.5 = 25.2
25.2 - 13.8 = 11.4

2. c) 61 

9*0.5 + 0.5 = 5
5*1 + 1 = 6
6*1.5 + 1.5 = 10.5
10.5*2 + 2 = 23
23*2.5 + 2.5 = 60
60*3 + 3 = 183

3. e) 188 

186 - 36 = 154
154 - 16 = 140
140 - 8 = 132
132 - 4 = 128
128 - 2 = 126
126 - 1 = 125

4. d) 151 

2*1 + 2 = 4
4*2 + 3 = 11
11*3 + 4 = 37
37*4 + 5 = 153
153*5 + 6 = 771
771*6 + 7 = 4633

5. b) 394 

391 + 2 = 393
393 + 6 = 399
399 + 12 = 441
441+ 20 = 431
431 + 30 = 461
461 +40 = 503

MATHS & REASONING BY - DR V K OMAR

Question 1: 353 354 351 356 349 ? 

a) 348 b) 358 c) 338 d) 385 e) 340 
Question 2: 1 5 13 29 ? 125 253

 a) 83 b) 69 c) 61 d) 65 e) 81 ...










No comments:

Post a Comment